Academic help online

Academic help online
Diagnostic Assessment

2

Question 1

A man and woman had been dating for a few years.

One day, the man decided to murder his father, in order

to obtain the inheritance that was promised him under

the father’s will. The man asked the woman to assist

him in the murder, and the woman agreed. The woman

subsequently purchased a knife for use in the murder.

However, the following day, the woman changed her

mind. She called the man and told him that she would

not participate in the murder. She also returned the

recently purchased knife to the store. She did not call

the police. The man later murdered his father and was

apprehended for the crime. The man’s criminal trial

concluded with a finding of not guilty.

Is the woman guilty of murder as the man’s accomplice?

(A) Yes, because she did not thwart the murder.

(B) Yes, because she did not effectively withdraw.

(C) No, because the man was found not guilty.

(D) No, because she effectively withdrew from

participation in the crime.

Question 2

A woman had lived in Apartment 123 in the same

apartment complex for six years. On many prior

occasions, the woman had hired the same handyman

to perform odd jobs in her apartment. The woman,

who was leaving town on vacation, telephoned the

handyman and said, “If you will replace the tile in

my bathroom while I’m on vacation, I will pay you

$700. You can get the keys from the superintendent.”

The handyman responded, “It’s a deal.” During their

telephone conversation, the woman failed to tell the

handyman she no longer lived in Apartment 123, but

had moved down the hall into Apartment 132.

The handyman came to the apartment complex, got the

key for Apartment 123 from the superintendent, and

re-tiled the bathroom. When the woman returned from

her vacation, she noticed that her bathroom tile had not

been replaced. She contacted the handyman, who then

informed her that he had replaced the tile in Apartment

123. The handyman demanded that the woman pay him

$700, but the woman refused.

If the handyman sues the woman for payment of the

$700 and the woman claims mistake, judgment

should be for whom?

(A) The handyman, because the woman should

have realized that he would replace the tile in

Apartment 123.

(B) The handyman, because even though no

contract existed, he is entitled to quasicontractual

relief under the circumstances.

(C) The woman, because the handyman did not

replace the tile in the woman’s bathroom.

(D) The woman, because no contract existed due

to the mutual mistake of the parties.

3

Question 3

A woman and her best friend went out to lunch at a local

restaurant. During their lunch, the friend received several

calls on her cell phone, and the woman grew increasingly

frustrated at the constant interruptions. When the friend

stepped out to use the ladies’ room, leaving her cell phone

on the table, the woman took the phone, turned it off and

put it in her purse, planning to return it at the end of lunch.

However, the woman forgot to return the phone to her

friend and ended up taking it home with her.

If the woman is charged with larceny, should she

be convicted?

(A) No, provided the woman eventually gave the

phone back to her friend.

(B) No, because the woman did not intend to

permanently deprive her friend of the phone.

(C) Yes, because the woman knowingly deprived

the friend of her phone.

(D) Yes, because the woman purposely took and

kept the friend’s phone.

Question 4

At the start of the basketball season, the team’s coach

decided not to start a player who was a long-time fan

favorite. As the team started to lose on a consistent

basis, one season ticket holder became disgusted with

the coach and began wearing a t-shirt suggesting that

people stop attending the team’s games. During the

games, the fan would walk in front of the TV camera

with his t-shirt slogan prominently displayed.

As the season progressed, many other fans joined in

heckling the coach and wearing similar t-shirts. Soon

thereafter, the fan received a notice from the team’s

management revoking his season tickets. The fan was

informed that he would not be allowed to attend any

more games, but would receive full reimbursement for

all remaining games.

If the fan brings an appropriate action against the

team challenging the revocation of his season tickets,

will he prevail?

(A) Yes, because he was entitled to express his

rights of free speech.

(B) Yes, because he had paid for the tickets for the

whole season.

(C) No, because his license to attend games

was revocable.

(D) No, because obscene speech is not

constitutionally protected.

Diagnostic Assessment

4

Question 5

An author sued a production company and a screenwriter

for violating his copyrights by producing an unauthorized

TV adaptation of his book. He filed suit in federal court,

suing both defendants in the Southern District of New

York. The author met with the production company, and

worked out a deal where the author’s name would be on

the show’s credits and he would receive a significant fee

as an advisor. Based on the settlement, the author decided

to dismiss his claim against both parties. Before the

author did so, the screenwriter, filed an answer, claiming

he co-owned the copyright based on a 20-year-old

contract with the author. In his answer, he stated several

times how much he was looking forward to seeing the

author in court and making the author spend a lot of

money in attorneys’ fees.

How can the author correctly dismiss his claim?

(A) By filing a notice of dismissal.

(B) By filing a stipulation.

(C) By making a motion for voluntary dismissal.

(D) By moving for summary judgment.

Question 6

A man was injured when he attempted to rescue his

friend from a lake, which he had fallen into during a

fight with a fisherman. The man sued the fisherman in

the appropriate U.S. District Court, but not his friend.

The fisherman claims that the friend’s conduct was the

exclusive cause of the incident that resulted in injury

to the man and seeks to join the friend in the action.

Under applicable state law, if the friend is not named

as a defendant and the friend and fisherman were joint

and several tortfeasors, the fisherman would have no

right of contribution from the friend if a judgment were

obtained against the fisherman. The man is a citizen

of Idaho, the fisherman is a citizen of Maine, and the

friend is a citizen of Idaho.

How should the Court rule on the fisherman’s motion

to join the friend in the action?

(A) The Court should grant the motion, because

the fisherman’s contention that the friend

was responsible for the incident may (at trial)

prove to be correct.

(B) The Court should grant the motion, because

there is a common nucleus of operative facts.

(C) The Court should deny the motion, because

the friend is not an indispensable party.

(D) The Court should deny the motion, because

joinder of the friend would destroy subject

matter jurisdiction.

5

Question 7

A high school sophomore had recently obtained her

driver’s license. One evening, she was driving home from

the library when a heavy rain began to fall. The rainstorm

completely washed out the roadway, and the girl’s car

got stuck in an embankment in a remote area. She exited

the car and began walking, hoping to be rescued by

another motorist. Miraculously, the girl came upon a

hotel. She entered the lobby and told the owner about her

predicament. Realizing that the girl had no place else to

stay, the owner told her that he had a room available and

that the room charge would be $300 per night. The owner

said this knowing that the standard room charge at the

motel was $200 per night. Due to the inclement weather,

the girl was forced to stay for two nights.

How much are the girl’s parents liable to pay the hotel

owner for their daughter’s two-night stay?

(A) Nothing, because parents are not liable for

necessaries furnished to a minor.

(B) Nothing, because contracts of minors are voidable.

(C) $400, which is the reasonable rate of

furnishing necessaries to their daughter.

(D) $600, which is the contract rate of furnishing

necessaries to their daughter.

Question 8

A man purchased a new bike. One day, he rode the new

bike to a local movie theater for the eight o’clock show

and parked his bike out front. As it was a small town,

he felt comfortable leaving his bike on the rack without

locking it. While the man was in the movie theater, the

ticket agent decided to take the bike out for a spin. The

ticket agent rode the bike once around the block and

placed it back in the rack just as the man had left it. As

the ticket agent was placing the bike back in the rack, the

handlebars gently rubbed against the rail. After the movie,

the man retrieved the bike and had no idea that it had been

moved, until he noticed a small scratch on the handlebars.

If the man files suit against the ticket agent, will he prevail?

(A) No, because the man was unaware of the

ticket agent’s use.

(B) No, because there was insufficient damage to

the man’s bike.

(C) Yes, for trespass to a chattel.

(D) Yes, for conversion.

Question 9

A bar manager told one of his bartenders that there

was a new type of home blender on the market that

could handle the ingredients of any drink, as well as the

ingredients to make salsa and other chopped foods. The

bar manager intended to wait in line to buy the blender

at a special sale, and the bartender asked the manager

to pick an additional blender up for the bartender as

well. The bar manager purchased three blenders and

kept one for himself. He sold one to the bartender for

the sale purchase price and gave the second to his sister

as a gift. Two weeks later, the bar manager came across

information on the Internet that the blender manufacturer

was recalling the blenders because the motors could

overheat and could throw sparks, causing a fire. The bar

manager called his sister right away and left a message,

but she didn’t get it in time. Her blender sparked, causing

a fire in her kitchen that burned her right hand and caused

major property damage. The bartender was also burned

by his blender, although he didn’t suffer nearly as much

property damage. The bar manager apologized to the

bartender for not having told him about the recall.

If the sister and the bartender file a strict products

liability action against the bar manager for damages,

what is the likely outcome?

(A) The bartender and the sister will prevail

against the bar manager, because the bar

manager is strictly liable.

(B) The bartender, but not the sister, will prevail

against the bar manager, because the bar manager

failed to warn the bartender about the recall.

(C) Neither the bartender nor the sister will

prevail against the bar manager, because the

bar manager is not a commercial seller.

(D) Neither the bartender nor the sister will

prevail against the bar manager, because the

bar manager did not tamper with the blender.

Diagnostic Assessment

6

Question 10

An engineer sued her employer in federal district

court alleging violations of False Claims Act, a federal

statute. The engineer filed her complaint and the

employer timely filed an answer. No other pleadings

were entered in the case. Discovery commenced.

One month after the employer’s answer was filed, the

engineer filed a written demand for a trial by jury.

Which of the following statements is correct?

(A) Because more than 14 days passed between the

filing of the last pleading and demand for trial

by jury, the case will not be tried by a jury.

(B) Upon proper motion, the court has the discretion

to grant relief and order a trial by jury.

(C) The Seventh Amendment requires that the

case be tried by a jury, regardless of the point

at which the demand is filed.

(D) The Sixth and Seventh Amendments require

that the case be tried by a jury, regardless of

the point at which the demand is filed.

Question 11

A married couple was waiting on line for tickets to

a movie. A man waiting ahead of them was quite

overweight. The husband made several comments

to his wife about how fat the man was, and the man

overheard. He confronted the husband and demanded

an apology. The husband refused. The man again

demanded an apology and threatened to punch the wife

if he did not get one. To show he was serious, the man

pushed the wife and waved a closed fist in her face.

Fearing that his wife would suffer a heart attack from

shock, the husband reached down, grabbed a nearby

pipe lying on the ground, and beat the man in the head

with it repeatedly. The man died from head injuries.

If the husband is prosecuted for criminal homicide,

should the court instruct the jury on voluntary

manslaughter in addition to murder?

(A) No, because the husband clearly intended to

kill or at least seriously injure the man.

(B) No, because the husband was the initial

wrongdoer, having provoked the man with

his rude comments.

(C) Yes, because the jury could reasonably

conclude that the husband lacked the intent

to kill or injure and was merely engaged in

reckless conduct.

(D) Yes, because the jury could find that the husband

unreasonably, but honestly, believed that deadly

force was needed to defend his wife.

7

Question 12

A woman loved to watch her neighborhood football

games. According to the neighborhood tradition,

whenever a player scored a touchdown, the player would

immediately rush full-speed at one of the fans on the

sidelines and jokingly tackle him or her. One Saturday,

as the woman and her neighbors are enjoying a game,

a player scores a touchdown. He immediately looks for

his little brother on the sidelines, intending to tackle

him. The woman, standing directly behind the player’s

brother, sees the player looking in their direction.

Exhilarated by the play, she jokingly calls out, “Come

on, big boy!” The player charges at her. Realizing that

she has become the player’s target, the woman ducks,

but the player succeeds in tackling her. The woman falls

to the ground and sustains a concussion.

If the woman now brings suit against the player for her

injuries, will she recover?

(A) No, because injuries sustained during sporting

events are foreseeable.

(B) No, because the woman consented to the

player’s contact.

(C) Yes, because the woman did not expressly

consent to the contact.

(D) Yes, because the woman suffered a harmful

and offensive contact.

Question 13

A tenant rented an apartment and entered into a written

lease agreement for a term of six years with a monthly

rental fee of $1,500. The tenant resided in the apartment

for three years. Then, the tenant drafted a written

agreement in which he transferred his entire interest

for two years to a friend. According to the written

instrument, the friend was obligated to directly pay the

tenant $2,000 per month for the term of his occupancy.

For the next six months, the friend paid the tenant

$2,000 each month, of which the tenant paid the

landlord $1,500. During the seventh and eighth

months, the friend continued to make his $2,000

payments to the tenant. However, the tenant did not

make any rental payments to the landlord for those two

months. After not receiving his rental payments, the

landlord went to the apartment and found the friend in

possession. The landlord then sued the friend for the

two months’ rent that had not been paid.

For whom should the court enter a judgment?

(A) The friend, because he made his monthly

rental payments to the tenant.

(B) The friend, because as a subtenant he is not

obligated to the landlord.

(C) The landlord, because the tenant transferred

his entire interest.

(D) The landlord, because there is privity of

contract between the landlord and the friend.

Diagnostic Assessment

8

Question 14

An elderly man’s house was frequently the target of

vandalism by local teenagers. Tired of having to repair

the damage they did to his home, the man decided to

lie in wait for teenagers on the night before Halloween,

when they often vandalized his home. True to form,

teenagers in costume appeared and began spray-painting

the man’s house. Wanting to scare the teenagers, the man

fired a shotgun over their heads. Some of the shotgun

pellets went through the top of a tall hat one of the

teenagers, who was dressed as Abraham Lincoln, but

the pellets did not harm him. The man was subsequently

arrested.

What, if any, crime did the man commit?

(A) Mayhem.

(B) Battery.

(C) Attempted murder, because a shotgun is an

inherently dangerous weapon.

(D) No crime.

Question 15

On Monday, three men agreed to rob a convenience

store that Friday morning. The next day, one of the men,

without notifying the other two, stole a truck to be used

as the getaway vehicle in the robbery. The day after that,

the man was arrested by the police for parole violations

and was placed in jail without bail. Nevertheless, the

other two men proceeded with their plan to rob the

store. One of the other men, however, was a police

informant who notified the police of the impending

robbery. The police waited at the store and arrested the

remaining man as he entered the store. Conspiracy in this

jurisdiction requires proof of an overt act in addition to

an agreement.

Of which of the following crimes may the man who stole

the truck properly be convicted?

(A) Theft of the truck only.

(B) Theft and either conspiracy to commit robbery

or attempted robbery, but not both.

(C) Theft and attempted robbery only.

(D) Theft, conspiracy to commit robbery, and

attempted robbery.

Question 16

An uncle wanted his nephew to quit smoking because

smoking is unhealthy. The uncle attempted to convince the

nephew to quit, but the nephew would not agree. The uncle

then offered to pay the nephew $5,000 if the nephew quit

smoking for one year. It was the uncle’s hope that if he could

get the nephew to stop smoking for one year, he would not

pick up the unhealthy habit again. The nephew agreed and

signed a contract to that effect with the uncle. The nephew

quit smoking for one year, but afterwards began smoking

again. He then asked for the $5,000, but the uncle refused to

pay. The nephew subsequently sues the uncle.

What is the likely outcome of the case?

(A) The uncle is liable, because there was a

bargained-for exchange.

(B) The uncle is liable, because the nephew relied

to his detriment on the uncle’s promise.

(C) The uncle is not liable, because the

consideration was inadequate.

(D) The uncle is not liable, because the uncle did

not receive a benefit and the nephew did not

suffer a detriment.

9

Question 17

A chef had equipped his home kitchen with the latest

cutting-edge appliances, including a brand-new

cappuccino machine. One night, when the chef was at

work and the nanny was at the chef’s home with the

chef’s two small children, the nanny decided to use

the cappuccino machine. The nanny knew that the chef

preferred that she not tamper with his equipment other

than to use the microwave oven to reheat the dinners

he prepared every day for his children. However, it was

a chilly night and the nanny was relatively certain that

she knew how to operate the machine. The nanny was

unaware, however, that the machine had been recalled

by the manufacturer for a design defect that caused

overheating of one of the handles and a risk of severe

burn. The chef knew of the recall but had not thought to

warn the nanny. In using the cappuccino machine, the

nanny burned her hand on the overheated handle and

had to seek medical treatment.

If the nanny files a strict products liability action against

the manufacturer of the cappuccino machine, is she

likely to prevail?

(A) Yes, if the nanny demonstrates that the defect was the

result of unreasonable design by the manufacturer.

(B) Yes, because the cappuccino machine was

defective when it left the manufacturer.

(C) No, because the nanny was not supposed to use

the chef’s equipment.

(D) No, because the nanny was not the purchaser of

the cappuccino machine.

Question 18

An Ohio corporation with its principal place of business

in Toledo sells appliances through independent dealers.

Each dealer has a separate agreement, which they

respectively executed with the corporation at the time

they were appointed to be dealers. Their agreements

entitle them to sell all appliances manufactured by the

corporation in their areas. This year, the corporation has

produced a new blender that represents a great advance

in blender technology. The corporation decides that

it will not sell this blender to its existing dealers but,

instead, will enter into agreements with new dealers to

handle the blender line. Twelve dealers in Indiana jointly

file actions for breach of contract, and an injunction

against the corporation in the appropriate U.S. District

Court. The corporation moves to dismiss the case for

lack of subject matter jurisdiction, and alternatively

moves to sever the actions.

How should the court rule on the corporation’s motions?

(A) The court should grant the corporation’s motion

to sever, because the claims arise out of the

same transaction or series of transactions, and

there are common issues of law and fact to all

of the claims.

(B) The court should grant the corporation’s motion

to dismiss, because the plaintiffs are not diverse.

(C) The court should deny the corporation’s motion

to sever, because plaintiffs are compelled to join

their actions if their claims arise out of the same

transaction or series of transactions.

(D) The court should deny the corporation’s motion

to dismiss, because the diversity requirement

has been met in this case.

Diagnostic Assessment

10

Question 19

An artist and a firefighter were neighbors who had been

feuding for some time. One afternoon, the artist found

some trash in her yard next to the fence that separated

her and the firefighter’s properties. Convinced that

the firefighter had deposited the trash on her property,

the artist picked up the trash and threw it onto the

firefighter’s property. The firefighter saw the artist

throw the trash onto her property and immediately

ran up to the artist with a sledgehammer in his hand,

got within inches of the artist’s face, and shouted, “If

you ever do something like that again I’ll break your

leg!” The firefighter then stormed off, leaving the

artist shaken by the encounter. The artist brought an

action against the firefighter for infliction of emotional

distress.

What is the likely outcome of the case?

(A) Judgment for the firefighter, because he did

not threaten any immediate physical harm.

(B) Judgment for the firefighter, as long as the

artist experienced no physical harm as a result

of the firefighter’s actions.

(C) Judgment for the artist, if the firefighter

intended that the artist experience

apprehension of a harmful physical contact.

(D) Judgment for the artist, if she suffered

severe emotional distress as a result of the

firefighter’s actions.

Question 20

A handyman owned a two-story hardware store on the

main street of town. The hardware store was located

on a corner lot, next to a retail building owned by a

lawyer. At the rear of the hardware store was a narrow

alley, wide enough to accommodate a car or small

truck. The handyman gave the lawyer oral permission

to use the alley behind the handyman’s store to access

the narrow space behind the lawyer’s building, where

the lawyer parks his truck every weekday. The lawyer

subsequently sold his building to a dentist, who

insisted on having an off-street parking space. At the

time of the conveyance, the lawyer negotiated with

the handyman and obtained an express easement in

favor of the retail building for a right-of-way through

the alley. The dentist took over the retail building

and parked behind the building on weekdays and

Saturdays. One day, however, the dentist’s vehicle was

vandalized, and he began parking in a secured lot.

Five years later, the dentist sold the building to a

pharmacy conglomerate that simultaneously also

purchased the hardware store from the handyman,

intending to renovate the two buildings into one

large retail space. Almost immediately thereafter,

the pharmacy conglomerate found a more attractive

location at which to carry out its plans, and it sold the

hardware store and retail building to two buyers. The

new owner of the hardware store placed a dumpster

in the alleyway, thereby blocking access to the space

behind the retail building. The new owner of the retail

building now seeks to park his car in this space.

If the new owner of the retail building seeks a court order

requiring the new owner of the hardware store to remove

the dumpster, what is the likely outcome of the case?

(A) The new owner of the retail building will prevail,

because an easement by prescription exists.

(B) The new owner of the retail building will prevail,

READ ALSO :   How is Yuan/Dollar exchange rate influenced by China’s national monetary policy (Senior Paper)

because the easement has not been abandoned.

(C) The new owner of the hardware store will

prevail, because the easement merged with

the fee when the pharmacy conglomerate

purchased both properties.

(D) The new owner of the hardware store will

prevail, because the new owner of the retail

building is estopped from asserting rights

under the easement.

11

Question 21

While hunting on public grounds, an outdoorsman

crossed over onto an adjoining private property.

The outdoorsman came upon a log cabin, which had

obviously been abandoned some time ago and was in

great disrepair. On inspection, the outdoorsman found

the cabin to be generally well built, except that the lock

on the front door was broken, the windows needed

to be replaced, and shrubs and bushes had nearly

overgrown the front porch. The outdoorsman cleared

out the overgrown bushes from the cabin, replaced the

lock on the front door, and installed new windows.

For the next year, the outdoorsman stayed in the

cabin nearly every weekend. After the first year, the

outdoorsman moved into the cabin. For the next 10

years, he lived in the cabin nine months out of the

year, leaving only during the harsh winter months,

as the cabin had no heating system apart from a

small fireplace, and snow and ice in the area made

it unsuitable for hunting or camping. During the

winter, the outdoorsman locked up the cabin and left

it unattended. No one other than the outdoorsman

ever occupied the cabin during that time period.

The outdoorsman had no word from the owner until

almost 15 years after his initial occupation, when one

Saturday morning, a man claiming to be the owner

showed up, pounding on the door and demanding that

the outdoorsman vacate the premises immediately.

The applicable statutory period for acquiring title by

adverse possession is seven years.

If the outdoorsman files an action to quiet title to the

cabin, will he prevail?

(A) Yes, because the outdoorsman was the only

person to occupy the cabin for a period of

more than ten years, and he held himself out

to be the owner.

(B) Yes, because the outdoorsman made

substantial improvements to the cabin.

(C) No, because the outdoorsman did not

possess the cabin continuously for the

statutory time period.

(D) No, because the outdoorsman took possession

of the cabin in bad faith, knowing that it

belonged to someone else, but intending to

claim it nonetheless.

Question 22

A buyer contracted in writing to purchase 200 Model-Y

widgets from a seller for $100 each. The contract

provided that the seller would deliver the widgets to

the buyer on or before June 1. The buyer and the seller

are both merchants. After entering into the agreement,

the seller discovered that the Model-Y widgets were

sold out. However, the seller did have a large quantity

of Model-Z widgets in storage. The Model-Z widget

was newer and more durable than the older Model-Y

widget. Both parties were aware that the retail price

for Model-Z widgets was $120 per unit. The seller

delivered 200 Model-Z widgets to the buyer and noted

that they were sent as an accommodation. These units

were received by the buyer on June 1.

If the buyer accepts the shipment of Model-Z widgets,

what will he be obligated to pay the seller?

(A) The contract price for the Model-Y widgets.

(B) The retail price for the Model-Z widgets.

(C) The fair market value for the Model-Z widgets.

(D) The reasonable price for the Model-Z widgets.

Diagnostic Assessment

12

Question 23

An apparel company ran a wholesale business selling

graphic T-shirts for skateboarders. Most of its products

were inexpensive T-shirts that resembled pricey

T-shirts sold in many specialty skateboard shops,

although they also sold licensed, branded T-shirts

from the popular skateboarding companies. In most

cases, the only way to tell the difference between

the inexpensive T-shirts and the original designs that

inspired them was to launder them to see whether they

maintained their size and bright colors.

A new skateboarding shop, wishing to order the authentic

skateboarding branded T-shirts for its discriminating

teen-aged customers, placed an order for 100 assorted

graphically designed T-shirts in all colors and sizes.

The apparel company mistakenly shipped 100 of the

inexpensive T-shirts, which looked the same, but were

not the licensed, branded T-shirts expected by the

skateboarding shop owner. When the skateboarding shop

discovered the mistake on the day of delivery, it demanded

that the licensed, branded T-shirts it had ordered be sent

instead. The apparel company refused, because it had just

received an order from a high-end skateboarding shop in a

resort town that would pay a premium price for all of the

authentic licensed, branded skateboarding T-shirts that the

apparel company had in stock.

In a breach of contract action by the skateboarding

shop against the apparel company, who will prevail?

(A) The apparel company, because there was no

meeting of the minds to form the contract.

(B) The apparel company, because its shipment

was a mere counteroffer, which the

skateboarding shop rejected upon delivery.

(C) The skateboarding shop, because it is the

master of the offer, not the seller.

(D) The skateboarding shop, because the shipment

of the wrong T-shirts was both an acceptance

and a breach by the seller.

Question 24

A buyer contracted with a seller to purchase Bill’s

SuperSports Bar, a popular establishment in a highvisibility

commercial area. The seller sent the buyer a

document via overnight express mail, which stated: “I,

Seller, agree to sell Bill’s SuperSports Bar to Buyer for

$850,000. Check for 10% down received, in escrow.

Signed, Seller. Date: Nov. 15, 1999.” The document

was also signed by the buyer directly below the seller’s

signature. The buyer responded by letter in which he

outlined the time and place of the closing: Jan. 3, 2000,

at 10 a.m. at the Shield Law Offices located in an office

complex down the street from the SuperSports Bar.

On the appointed day, both parties duly appeared to pass

title to the property. The seller produced a deed conveying

Bill’s SuperSports Bar to the seller from another owner

dated Dec. 28, 1999; the buyer learned that the other

owner, in fact, owned Bill’s SuperSports Bar on Nov. 15,

1999. The buyer refused to perform the contract, arguing

that the seller cannot give good title to the property.

In a suit by the seller for specific performance, what is

the likely outcome?

(A) The buyer will prevail, because he did not

contract with the other owner to buy the

SuperSports Bar.

(B) The buyer will prevail, because the seller did

not hold marketable title on Nov. 15, 1999.

(C) The seller will prevail, because he fulfilled

the terms of the agreement by delivering

marketable title on January 3.

(D) The seller will prevail, because the buyer

knew that another owner owned the

SuperSports Bar at the time the parties

entered the contract.

13

Question 25

After he was turned down for a promotion, a man

decided to murder his boss. The man purchased a gun

for this purpose and then invited his boss to his house

for dinner. However, the day before the boss arrived

at the man’s house, the man changed his mind and left

the gun under his mattress. The man was subsequently

charged with attempted murder.

Should the man be convicted or acquitted?

(A) The man should be acquitted, because the

boss was unaware of the man’s motives.

(B) The man should be acquitted, because his actions

were not sufficient to constitute an attempt.

(C) The man should be convicted, because a

person is presumed to intend the natural and

probable consequences of his acts.

(D) The man should be convicted, because he

purchased the gun with the intent to kill his boss.

Question 26

A landowner conveyed a large estate “to my daughter,

her heirs and assigns, on condition that she produce an

heir with her husband, but if she dies without such an

heir, then to my personal assistant.” Five years after

the landowner conveyed the estate to her daughter, the

personal assistant and the landowner got into a serious

dispute, which resulted in the personal assistant’s

decision to set up her own rival business. At the same

time, the landowner’s daughter and her husband

divorced. Three years later, the husband died of a

stroke brought on by undiagnosed high blood pressure.

The daughter remarried to a new business protege of

her father. The estate was a heavily forested property

with a number of stands of maple trees. The daughter

and her new husband walked the property and marked

stands to be harvested for wood. The daughter then

exploited the timber on the estate for a large profit.

Was the daughter’s act of exploiting the timber for

profit permissible?

(A) No, as affirmative waste, because the daughter

was only a life tenant of the estate.

(B) No, as trespass, because the personal assistant

owns the estate in fee simple absolute.

(C) No, because the daughter has not produced an

heir with her first husband.

(D) Yes, because the daughter holds in fee simple.

Question 27

A safety inspector was injured when the conveyor

belt he was inspecting malfunctioned. The belt was

located in a factory owned by a Utah corporation

with its principle place of business and headquarters

located in Washington. The inspector brought a cause

of action against the corporation in Federal District

Court in Washington. Service of process was sent to

the residence of the corporation’s president. Service

was accepted by the president’s 18 year old son. The

corporation filed two motions. The first motion, made

one week after process was served, was made under

Rule 12(b)(1) for lack of subject matter jurisdiction.

The second motion, filed a day later, was brought under

Rule 12(b)(5) for insufficient service of process. The

court ruled that the corporation had waived its right to

complain about service of process.

Was the court’s action proper?

(A) No, because service of process must be made

to the agent for service of process.

(B) No, because the motion was made within 20

days of service of process.

(C) Yes, because the son was 18 years old and had

the capacity to accept service of process.

(D) Yes, because the corporation waived its right

to object to service of process.

Diagnostic Assessment

14

Question 28

Five years ago, an architect, a citizen of State X, designed

a building for a State Y corporation which had its

principal place of business in State Y. Six months ago, the

building’s mezzanine collapsed in causing damages in

excess of $100,000. The corporation has filed an action

against the architect in the U.S. District Court in State X

alleging that damages resulted from the negligent design

of the building. The architect filed an answer denying

negligence and asserting as an affirmative defense that

the statute of limitations for negligence is two years.

The architect filed a motion for summary judgment

accompanied by an affidavit stating that the building had

been constructed six years ago. Although the statute of

limitations on negligence claims in State X is two years,

the architect’s motion was denied.

Was the architect’s motion for summary judgment

properly denied?

(A) Yes, because the accident occurred within the

past 2 years.

(B) Yes, because the architect has the burden of proof

with respect to its statute of limitations defense.

(C) No, because the corporation failed to file a

responsive affidavit.

(D) No, because the building was constructed six

years ago.

Question 29

The annual month-long wine festival was in full force

in the small coastal community in which a 14-yearold

teenager lived. During the days, celebrants winetasted,

shopped, and ate various delicacies at the small

wineries that lined the community’s ten-block Main

Street. Evenings brought more drinking, along with

dancing and eating until midnight each night. To keep

Main Street clear for maintenance vehicles to prepare

the festival area for the next day’s activities, the coastal

community had enacted a ban on motorized vehicles

between the hours of 2:00 a.m. and 8:00 a.m.

One morning during the festival at 7:30 a.m., two

women had just finished breakfast at a cafe on Main

Street when they walked out onto the street to enjoy the

morning sun. Just at that moment, the teenager rounded

the corner onto Main Street riding a motorized bicycle.

The teenager was running late completing his morning

paper route. The motorized bicycle the teenager was

riding had a top speed of only 15 miles per hour, and the

teenager would have been able to stop in time except

that the bicycle was out of brake fluid. The teenager had

owned the bicycle for two years but had never examined

the brakes. The brakes failed and the teenager hit one of

the women, knocking her unconscious.

If the woman files suit against the teenager, what is the

probable outcome?

(A) The woman will prevail, because the teenager

was negligent per se.

(B) The woman will prevail, if a reasonably

prudent child of the same age and experience

would have known the necessity to service the

brakes of the motorized bicycle.

(C) The teenager will prevail, because the

woman’s injury was not the type of harm the

statute was designed to prevent.

(D) The teenager will prevail, because a

manufacturing defect was the cause of the

woman’s injury.

15

Question 30

An eighty-year-old farmer had a successful cattle farm.

The farmer had little family and wanted to ensure that

the farm would continue to be operated by someone

who appreciated the farm’s history and enjoyed working

with cattle. The farmer had only one son who was 45

years old and had no children yet. One of the farmer’s

employees had started working on the farm during

high school. The employee had shown a great interest

in cattle and was a loyal and hardworking employee.

That employee was in his early twenties. To ensure that

the farm would continue operating after his death, the

farmer included a clause in his will that provided that

upon the farmer’s death, the farm would go to his son

for life, and then upon the son’s death, the farm would

pass to the son’s children. The will further provided that

if the son had no children at the time of his death, then

the farm would pass to the valued employee. The farmer

died two years later, at which time his son still had no

children. The employee was still working on the farm.

Does the employee have a remainder interest in the farm?

(A) No, the employee has a fee simple interest

subject to a condition subsequent.

(B) No, the employee has an executory interest.

(C) Yes, the employee has a contingent

remainder interest.

(D) Yes, the employee has a vested remainder

interest subject to open.

Question 31

A company was engaged in transporting liquids. As is

normal, the company was hired to deliver a container

of highly flammable chemicals to a local plant. The

company required all of its employees to engage in

extensive training. On the day of the delivery, the

driver noticed a stalled car ahead of him in the right

lane. There was no way for the driver to avoid the car.

As the driver tried to maneuver around the car, some

of the chemicals spilled out of the container and ended

up on the car. The driver, who had a cigarette in his

mouth, quickly caught fire and later died. The driver’s

estate brings a strict liability action.

Will the company be liable for the injuries caused

to the driver?

(A) No, because the truck driver and the company

exercised due care.

(B) No, because the car’s driver should have

removed the car from the road.

(C) Yes, because transporting flammable liquid is

an abnormally dangerous activity.

(D) Yes, because transporting flammable liquid

is illegal.

Diagnostic Assessment

16

Question 32

A man went to see a play at the local theater operated by

a local theater company. The man bought a ticket which

assigned him to sit in Seat 24 in Row 30. When the

man handed his ticket to the usher the usher glanced at

the ticket and lead the man to what he believed was the

appropriate seat. However, the usher mistakenly took the

man to Seat 23 in Row 30. The man occupied Seat 23 in

Row 30, failing to notice it was not the seat listed on his

ticket. During the second act of the play the seat which

the man was occupying suddenly collapsed, causing the

man several injuries. A subsequent investigation failed to

determine why the seat collapsed.

If the man brings an action for negligence against the

theater company, which of the following is the most

accurate statement?

(A) In order to prove breach of due care, the man

must show that the usher acted unreasonably

in leading him to the wrong seat.

(B) Under the doctrine of res ipsa loquitur, the fact

that the seat collapsed is sufficient to establish that

the theater company breached its duty of care.

(C) In order to prove that the theater company

breached its duty of care, the man must

demonstrate why the seat collapsed.

(D) Because the investigation failed to determine

why the seat collapsed, the man cannot

prevail in a negligence action against the

theater company.

Question 33

A graphic designer, who was a citizen of New York,

filed an action in the U.S District Court in Brooklyn

against an Oklahoma corporation and a New Jersey

corporation, alleging that they infringed his trademarks

in violation of the Lanham Act, a federal statute,

and that his actual damages from the conduct of the

defendants was $73,000. The designer further alleged

in a second cause of action that the New Jersey

corporation owes him $1,000 for graphic design work.

Which of the following is true?

(A) The action should be dismissed by the

District Court, because the value of the

aggregated claims does not meet the amount

in controversy requirement.

(B) The District Court may exercise pendent

jurisdiction over the second cause of action,

but only if it arises from a common nucleus of

operative facts with the first cause of action.

(C) The federal court has subject matter jurisdiction,

because there is complete diversity of citizenship

between all plaintiffs and defendants.

(D) The federal court lacks subject matter

jurisdiction, because Lanham Act violations

must be tried in the state court where such

violations allegedly occurred.

Question 34

A man conveyed his property via quitclaim deed to his

sister. Two months later, his sister was evicted from

the property by a neighbor with superior title. The

neighbor’s interest was unknown to the man, because

this interest had been concealed by the person who

sold the property to the man. The sister now brings suit

against the man for damages.

Should the sister prevail in her action for damages?

(A) Yes, because the eviction violates the

covenant against encumbrances.

(B) Yes, because the eviction violates the

covenant of quiet enjoyment.

(C) No, because the deed by which the man

conveyed the property to the sister was not a

special warranty deed.

(D) No, because the man conveyed the property to

the sister via quitclaim deed.

17

Question 35

A pedestrian sued a driver in state court in a state that

follows the Federal Rules of Civil Procedure. The pedestrian

seeks to recover for negligence for personal injuries

resulting from being hit by a car driven by the driver. The

driver simply denied the pedestrian’s allegations. The

pedestrian demanded trial by jury. Along with the forms for

a general verdict, the court submitted written interrogatories

to the jury. In answering these interrogatories, the

jury found the following: that the driver had hit the

pedestrian with his car; that the driver negligently hit the

pedestrian; that the driver caused the pedestrian damages

as alleged in the complaint. The jury also returned a

judgment against the pedestrian. The judge then ordered

a new trial. The pedestrian then filed a stipulation of

voluntary dismissal, which was also signed by the driver.

Which of the following statements is correct?

(A) The pedestrian may later bring an action

against the driver on the same facts.

(B) Where the jury’s answers to interrogatories

conflict with a general verdict, the judge must

enter a verdict in conformity with such answers.

(C) Where a jury’s answers to interrogatories conflict

with a general verdict, the judge must either enter

the general verdict or order a new trial.

(D) Because the pedestrian filed a stipulation

of voluntary dismissal, he cannot initiate a

subsequently similar action against the driver

in the future.

Question 36

During trial, the plaintiff demanded that the defendant

produce certain documents regarding transactions that

had occurred at the plaintiff’s brokerage. The defendant

claimed the documents were protected by the attorneyclient

privilege. The court ordered the documents to be

produced. The defendant immediately filed an appeal

in federal appeals court challenging the court’s order

regarding the documents.

May the defendant do so?

(A) No, because this is an interlocutory order.

(B) No, because there is no indication the judge

abused her discretion.

(C) Yes, because without an appeal, the

documents will be revealed and privilege will

be lost forever.

(D) Yes, because this is a final adjudication.

Question 37

On February 1, a woman contracted with a famous

landscaper to drastically change the sprawling front

lawn of her mansion. The woman had long admired

the landscaper’s work. The landscaper promised to

complete the work before April 30, the date of the

woman’s annual spring party.

On March 1, the woman left for a month’s vacation.

After beginning the work on the woman’s lawn,

the landscaper became increasingly anxious about

finishing other prior commitments and assigned the

contract to a local lawn care professional. When the

woman returned, she was horrified to find the lawn care

professional shaping the hedges on the front lawn.

The woman contacted the landscaper, and he informed her

of the assignment to the lawn care professional and told

her that he was no longer obligated under their original

contract. Although the landscaping project was coming

along nicely, the woman fired the landscaper anyway.

If the woman asserts a claim against the landscaper

based on breach of contract, what effect will the fact

that the landscaper assigned the contract to the lawn

care professional have?

(A) It will relieve the landscaper of liability,

because the contract was assignable.

(B) It will relieve the landscaper of liability,

because the landscaping project was

satisfactory to the woman thus far, and she

suffered no damages.

(C) It will not relieve the landscaper of liability,

because a personal services contract of this

nature is non-delegable.

(D) It will not relieve the landscaper of liability,

because the landscaper did not inform the

woman of the assignment in advance.

Diagnostic Assessment

18

Question 38

A grantor held record title to a ten-acre parcel of

land. The grantor conveyed the land to a grantee by

warranty deed, which the grantee failed to record.

The grantee took possession of the land and built a

fence around the property. He also installed a sign at

the entrance to the property which read: “Welcome

to our little piece of heaven.” The sign included the

grantee’s full name, and the purchase date.

The following year, the grantor executed a special

warranty deed purporting to convey the same ten-acre

parcel of land to a developer for valuable consideration.

This deed was promptly recorded by the developer.

Before this transaction was consummated, the developer

inspected the property and was aware of the grantee’s

ownership claim. When the developer inquired about the

grantee, the grantor truthfully told him that the grantee

had never recorded the earlier deed. Thereafter, the

developer conducted his title search and verified that the

deed from the grantor to the grantee was unrecorded.

Two weeks after the developer’s recordation, the grantee

learned of the grantor’s conveyance to the developer and

then recorded his deed.

This jurisdiction has the following recording statute

in effect:

“An unrecorded conveyance or other instrument is

invalid as against a subsequent bona fide purchaser for

value, without notice, and who records first.”

In a suit between the grantee and the developer to quiet

title to the land, who will prevail?

(A) The developer, because in accordance with the

doctrine of first-in-time, first-in-right, his deed

was recorded earlier in time.

(B) The developer, because as a subsequent bona

fide purchaser, he is given priority under the

recording statute.

(C) The grantee, because his physical possession

and fencing of the property provided notice of

his ownership claim.

(D) The grantee, because the grantor’s conveyance

to the grantee preceded the purported property

transfer to the developer.

Question 39

A young woman felt crushed after her boyfriend of

two years broke up with her. Feeling despondent, she

consumed several bottles of wine, and then decided it

would be a good idea to fry up some bacon. Once she

got the bacon frying, however, the woman lost interest

in the food and sat on her couch to watch television.

Within minutes, the woman passed out, leaving her

food burning on the stove. Before long, the fire alarm

went off, and the apartment filled with smoke.

Luckily, the woman’s neighbor called 911. Within a

few minutes of the call, the fire department arrived at

the woman’s house. However, because the smoke was

so thick inside the house, one of the firemen suffered

READ ALSO :   Synthesis of Carboxylic Acids

smoke inhalation and had to be taken to the hospital.

If the fireman asserts a claim against the woman to

recover for his injuries, will he prevail?

(A) No, because the woman did not intend to

harm the fireman.

(B) No, because the firefighter’s rule treats

emergency personnel as licensees.

(C) Yes, because the woman intentionally placed

herself in a position of peril.

(D) Yes, because the fireman succeeded in saving

the woman’s life.

19

Question 40

A woman was hit by a train and injured when she

drove through a railroad crossing. She sued the train

company in federal court, claiming that the company

was negligent because its safety barriers failed to come

down and block passage through the crossing as the

train approached. At the jury trial, the woman produced

only one eyewitness, who said the barriers were not

down when the plaintiff approached the crossing.

During cross examination, the witness admitted that

she had very poor vision and could only see about 10

feet in front of her. At the time of the accident, the

witness was standing 200 feet away at an ATM, and her

view was obstructed by a large stand of pine trees. The

woman then rested her case. The company had several

witnesses from the nearby train station who would

testify that the barriers were in fact down.

What should the defendant do at this point?

(A) Move for summary judgment.

(B) Move for judgment as a matter of law.

(C) Counter the plaintiff’s contentions with its

own witnesses, who were all standing at the

nearby train station and would testify that the

barriers were down.

(D) Request dismissal.

Question 41

A man devised a 50-acre tract of land to his friend for life,

and then to the man’s son. The friend wanted to turn the

weekend cabin on the property into a year-round home.

To do so, he took out a construction mortgage with a bank

to pay for the remodeling. The bank properly recorded the

mortgage. After the cabin had been improved, but before

the mortgage had been paid off, the friend died.

If the bank and the man’s son both claim ownership of

the property, how should the court rule?

(A) In favor of the son, because a mortgage on a

life estate is void after the termination of the

life estate.

(B) In favor of the son, because the friend was

prohibited from mortgaging the property.

(C) In favor of the bank, because it properly

recorded the mortgage.

(D) In favor of the bank, because recording

statutes only affect bona fide purchasers.

Question 42

A disgruntled employee decided to kill his boss. The

employee planned to hide near his boss’s house and

shoot the boss when he arrived home from work. The

employee positioned himself behind a small grove

of trees about 500 yards away from the boss’s home.

Shortly thereafter, the boss pulled into his driveway.

As the boss exited his vehicle, the employee fired a

single shot from his rifle, but the shot did not reach the

boss. A neighbor heard the gunfire, however, and called

the police, who later arrested the employee as he was

fleeing the scene with his rifle. At the employee’s trial

for attempted murder, it was conceded, following the

testimony by a ballistics expert, that it would have been

impossible for the employee to hit his boss from that

distance with the rifle he had used.

Is the employee guilty of attempted murder?

(A) No, because at that distance he lacked the

requisite intent to kill his boss.

(B) No, because it was legally impossible for the

employee to kill the boss from that distance.

(C) Yes, because it was merely factually

impossible for the employee to kill the boss

from that distance.

(D) Yes, because the employee’s actions

were premeditated.

Question 43

An elderly man who lived by himself started

repainting his house. Because the elderly man was

having a difficult time with the project, his son hired a

contractor to do the job. Their contract provided that

the contractor would receive payment after completion

of the work. When the contractor was halfway done

with the job, the elderly man died.

Which of the following is correct?

(A) The contract is discharged, and the contractor

will receive no payments, because he did not

complete the work.

(B) The contract is discharged, but the contractor

will receive payment for work completed.

(C) The contract is not discharged, but the

contractor must receive half of the payment

before continuing work.

(D) The contract is not discharged, and the

contractor must complete the repainting project.

Diagnostic Assessment

20

Question 44

On January 1, a seller received an order in the

mail from a buyer requesting shipment of its latest

computer at the seller’s current catalog price. The

order specified that the seller could accept the offer

either by shipping the goods or promptly notifying

the buyer of acceptance. On January 2, the seller

mailed to the buyer a letter accepting the order and

assuring the buyer that the computer would be shipped

on January 3. On January 3, the seller realized that

he was out of that computer model and shipped to

the buyer a different computer model and a notice of

accommodation. On January 5, the buyer received the

seller’s January 2 letter and the different computer

model, but not the notice of accommodation.

Which of the following best states the parties’ legal

rights and duties?

(A) The buyer can either accept or reject the

different computer model and in either event can

recover damages, if any, for breach of contract.

(B) The buyer can either accept or reject the different

computer model, but if he rejects it, he will

thereby waive any remedy for breach of contract.

(C) The seller’s prompt shipment of nonconforming

goods constituted an acceptance of the buyer’s

offer, thereby creating a contract for sale of the

replacement computer model.

(D) The seller’s notice of accommodation was

timely mailed and his shipment of the different

computer model constituted a counteroffer.

Question 45

A woman was sitting on a bench in the park reading a

book. A father and his six-year-old daughter sat down

on the other end of the park bench. A bee suddenly

stung the little girl, who was very allergic to bees. The

father panicked because he could not find the bee sting

antidote, EpiPen, in the bag the mother had packed. The

woman ran to a nearby playground and asked if anyone

there had an EpiPen. A nanny told him that she did have

one but that she needed it in case the little boy she cared

for was stung by a bee. The woman explained that a

little girl had just been stung and could go into shock

and die without a shot soon. The nanny refused to hand

over the EpiPen, and the woman shoved the nanny to

the ground, grabbed the EpiPen from the nanny’s diaper

bag, and injected the little girl, saving her life.

Should a jury convict the woman of battery?

(A) Yes, because the woman was not related to the

little girl.

(B) Yes, because she intentionally applied

unlawful force to the nanny’s person.

(C) No, because the nanny, as a care-giver, owes

a special duty to others and her inaction

prevents her recovery.

(D) No, because the woman reasonably believed

that shoving the nanny was necessary to save

the little girl’s life.

21

Question 46

A homeowner owned a plot of land that he inherited

from his mother. Both the homeowner and his neighbor

often walked along a ten-foot strip of land on the edge

of the homeowner’s land to get to the nearby lake. The

neighbor frequently took this route to the lake because

otherwise he would have had to walk around the

homeowner’s property. For 23 continuous years, this

strip of land was used by both the homeowner and the

neighbor. The homeowner knew of, but did not consent

to, the neighbor’s use. Recently, the homeowner

decided to build a fence across his property in order

to prevent anyone else from using the strip to get to

the lake. The statutory period for prescription in the

jurisdiction is 20 years. The neighbor filed suit to

enforce his right of access to the easement.

Should the neighbor prevail?

(A) Yes, because the neighbor used the strip of

land without the homeowner’s permission.

(B) Yes, because the neighbor has acquired an

easement by prescription.

(C) No, because the neighbor’s use of the strip of

land was not exclusive.

(D) No, because the neighbor’s use of the strip of

land was not reasonably necessary.

Question 47

On January 10, a merchant seller mailed a letter to a

merchant buyer offering to sell 500 pairs of designer

jeans for $50,000. The buyer received and read the letter

on January 13. On January 14, after having carefully

considered the terms, the buyer dropped into his mailbox

a letter to the seller accepting the offer. That same

evening, the buyer went to retrieve his mail and found

a second letter from the seller. Too tired to read his mail

that evening, the buyer did not read the second letter until

January 15. The second letter, as it turned out, was a letter

revoking the offer, sent by the seller on January 11.

Since that time, the seller has sold the jeans to someone

else, and the buyer files suit for breach of contract.

For whom should the court rule?

(A) The buyer, because he dispatched his letter

before receiving the revocation.

(B) The buyer, because the seller made a firm offer.

(C) The seller, because he sent his revocation

before the buyer sent his acceptance.

(D) The seller, because the buyer’s failure to read

his mail upon receipt operates as a waiver.

Question 48

A day trader made several jokes about a lawyer’s

physical appearance at a bar. In an attempt to put an

end to the insults, the lawyer walked over to the day

trader, picked up a beer bottle and stared down the

day trader. Unaffected by the lawyer’s display, the day

trader laughed and sat back down.

Is the lawyer liable for assault?

(A) Yes, because the lawyer intended to put the

day trader in fear of a harmful touching.

(B) Yes, because the lawyer picked up a beer

bottle and approached the day trader.

(C) No, because the day trader was not in fear of

harmful contact.

(D) No, because the lawyer did not threaten an

immediate contact.

Diagnostic Assessment

22

Question 49

Late one evening, a teenage boy and his 15-year-old

girlfriend were just leaving the library. As they were

walking through a dimly lit parking lot to get to their

bikes, an adult man, who was visibly intoxicated, emerged

from behind a parked car and ran up to them. The man

knocked the boyfriend to the ground and hit him over

the head with a wrench, causing the boyfriend to lose

consciousness. The man then forced the girlfriend into his

car. Moments later the boyfriend regained consciousness,

and the man fled the scene. The man was subsequently

charged with assault with intent to commit rape.

Which of the following would provide the defendant

with his best defense?

(A) The defendant thought the girl had consented.

(B) The defendant did not intend to rape the girlfriend.

(C) The defendant’s intoxication at the time

negated the required general intent.

(D) It is impossible to prove that the defendant

was the perpetrator because the parking lot

was dimly lit.

Question 50

A man owned a ten-acre parcel of land with a farmhouse

and a barn, which he converted into a three-car garage.

The man entered into a three-year written lease agreement

with a tenant. The lease agreement provided that the

tenant would be entitled to possession of the farmhouse,

barn, and ten-acre parcel. The lease also contained an antiassignment

clause and a provision prohibiting subletting

without the landlord’s written consent.

Which of the following would be a permissible use

of the property?

(A) The tenant allowed his cousin to park his antique

car in the garage for the sum of $50 per month.

(B) During the summer months, the tenant

permitted his friends to swim and operate

motor boats on a lake that was situated on the

land leased from the man.

(C) The tenant allows a neighbor to come onto the

property and remove large amounts of topsoil,

which he uses in his landscaping business.

(D) The tenant allows his cousin to stay at the

house while on college break in exchange for

$25 per week.

Question 51

A female horse won the most competitive horse race in

the United States three years ago. Since her retirement,

her owner has not been able to successfully breed her.

A veterinarian who specializes in new techniques for

breeding animals that have not been able to conceive

approaches the horse owner with an offer to purchase

the female horse, explaining that the veterinarian has

always been a devotee of horse racing and has had a

lifetime ambition to own a winning horse. While the

veterinarian’s explanation is truthful, the veterinarian

does not tell the owner that the veterinarian wishes to

try this technique on the female horse.

The owner sells the female horse to the veterinarian

for $50,000, a generous price for a sterile, retired

racehorse, but far less than the value of the horse if

she could bear offspring. Soon after the veterinarian

purchases the female horse, the veterinarian extracts

eggs from her ovary, fertilizes them in a test tube, and

implants the fertilized eggs in her womb.

When the owner learns that the female horse is

pregnant and the sire is another winner of the most

competitive horse race in the United States, the owner

sues the veterinarian to rescind the sale.

In that action, judgment for whom?

(A) The owner will prevail, because his ignorance

of the fact that the female horse was capable

of being bred successfully constitutes a

unilateral mistake.

(B) The owner will prevail, because there was a

mutual mistake.

(C) The veterinarian will prevail, because attempts

to breed the animal by the veterinarian would

be foreseeable to the owner.

(D) The veterinarian will prevail, because the

owner has no legal basis on which to rescind

this valid contract.

23

Question 52

A male and female teenager wanted to go to a movie,

but neither had enough money. They went into a local

coffee shop and saw that there was a tip jar on the

counter. The female teenager said, “I bet there’s enough

money in the tip jar for us to go to a movie and get

some pizza afterwards.” The male teenager agreed and

suggested that the female teenager create a diversion

so that he could take the tip jar. The female teenager

agreed and said she would meet the male teenager a

few blocks away. The female teenager ordered a coffee

and when it came, she purposefully knocked it over

onto the barista. While the female teenager helped

the barista clean up, the male teenager grabbed the tip

jar and ran out of the coffee shop. As defined in this

jurisdiction, the crime of conspiracy requires an overt

act in furtherance of the conspiracy.

If prosecuted, the female teenager should be found

guilty of which of the following?

(A) Conspiracy, but not larceny.

(B) Larceny, but not conspiracy.

(C) Both larceny and conspiracy.

(D) Only larceny, because the conspiracy will

merge into the completed crime.

Question 53

The plaintiff, a citizen of West Virginia, filed suit

against the Bank of Ohio (“Bank”) in the U.S. District

Court for the Northern District of Ohio. Bank has its

only place of business in Ohio. Plaintiff seeks an order

directing Bank to deliver the proceeds of a savings

account in the amount of $105,000 to the plaintiff. She

alleges an agreement between her and McElroy, also

a West Virginian citizen, whereby they each deposited

an equal amount of money in the account to be held

solely in McElroy’s name. She further alleges that

McElroy has asked Bank to split the account equally

and to deliver a passbook to her but that Bank refused

to do this. Bank answers by alleging that it refused to

make the transfer, because McElroy claims that the

plaintiff assigned her interest in the account to him as

repayment for a loan. Bank makes a motion to dismiss

for failure to join McElroy as a party. At a hearing on

this issue, the Court orders joinder of McElroy and

then dismisses for lack of complete diversity.

Which of the following statements is correct?

(A) Plaintiff will not be barred from refiling the

action in a state court, because the dismissal

on the grounds provided in Federal Rule of

Civil Procedure 12(b)(7) does not constitute

an adjudication on the merits.

(B) The Court should not have required the joinder

of McElroy, because it destroyed diversity.

(C) The Court should not have dismissed the

action, because it has pendent jurisdiction

over the plaintiff’s claim against McElroy.

(D) The Court should not have dismissed the

action, because there was diversity between

the plaintiff and Bank.

Diagnostic Assessment

24

Question 54

A young woman obtained used items of clothing from yard

sales, swap meets and similar sources, and then cleaned

and refurbished them before selling them at her small

shop. One day, an extremely well-to-do lawyer came into

the woman’s shop. The lawyer found nothing of interest in

the clothing racks, but she noticed an odd-looking canarycolored

raincoat hanging from a hook on the wall.

Not realizing that the raincoat belonged to the young

woman, the lawyer pointed to the raincoat and said,

“I’ll give you $25 for that hideous thing.” Offended at

the lawyer’s condescending tone, the young woman

replied, “That raincoat isn’t worth a thing. Besides,

you’re far too corporate to wear a coat like that.” The

lawyer, now angry herself, replied, “I must have it.”

The young woman said, “Okay, it’s yours for $550.”

“Done,” said the lawyer.

Does an enforceable contract exist between the woman

and the lawyer?

(A) Yes, because there was a bargained-for

exchange of promises.

(B) Yes, because the raincoat was worth $550 to

the lawyer.

(C) No, because the consideration supporting the

woman’s promise is insufficient.

(D) No, because the agreement between the

parties is unconscionable.

Question 55

A child’s elementary school holds an annual Honeybee

Festival, where all of the children spend the day

learning about bees through hands-on experience.

One of the children’s mothers objected to the festival

because her son was allergic to bee stings and could die

if not treated in time. The mother asked the school to

not hold the festival, but the school refused to cancel it.

The mother sued in a state court of general jurisdiction,

in a state that followed the Federal Rules of Civil

Procedure. Because the festival was scheduled for 10

days later, she asked for a preliminary injunction that

required the school to hold the festival without any live

bees. The court denied the request.

Can the mother appeal this decision?

(A) Yes, because the grant or refusal of an

injunction is immediately appealable.

(B) Yes, because there was a final judgment.

(C) No, because the mother must wait until trial.

(D) No, because this was an interlocutory ruling.

Question 56

A man approached his neighbor, who was seven months

pregnant, and said, “If you name your son after my

father, I will pay you $200 per month for three years.”

The neighbor agreed and named her child after the man’s

father. Afterwards, the man reneged on his promise and

told his neighbor that he had decided not to pay her the

money. The neighbor sued the man for breach of contract.

Judgment should be for whom?

(A) The neighbor, because there was a bargainedfor

exchange.

(B) The neighbor, because the neighbor’s promise

to name the child after the man’s father was

a condition of a gift made in consideration of

carrying out a moral obligation.

(C) The man, because the man’s promise to

pay the neighbor was a mere gift promise

unsupported by consideration.

(D) The man, because the neighbor did not

experience any recognizable detriment in the

naming of her child.

25

Question 57

A manufacturer of single serving coffee makers mailed

to a retailer of various household appliances the

following written offer: “Will sell 50 single serving

coffee makers at our list price of $50 each. Available

for immediate delivery. Please respond by return mail.”

Unaware of the manufacturer’s offer, the retailer, on the

same day, mailed a written letter to the manufacturer

containing the identical terms: “Will purchase 50 single

serving coffee makers at your list price of $50 each.”

Both letters were mailed through the U.S. postal system.

Before either party has received the other’s correspondence,

which of the following accurately states the legal

relationship between the manufacturer and retailer?

(A) Because there was a meeting of the minds,

a contract was formed when the retailer

mailed its letter.

(B) A contract exists between the parties, with

UCC gap-filler provisions controlling for

delivery and payment terms.

(C) A contract was formed when the retailer

deposited its acceptance, because the crossing

communications contained identical terms.

(D) No contract exists.

Question 58

A businessman asked to see an expensive watch in

a jewelry store. In conversation with the clerk, the

businessman falsely claimed to be the son of the city’s

district attorney. When handed the watch, he asked if he

could put it on, walk around a bit so he could see how it

felt on his wrist, and then briefly step outside to observe

it in natural light. The clerk agreed, saying, “I know I

can trust someone like you with the merchandise.” The

businessman walked out of the store wearing the watch

and never returned. A week later, the clerk was at a

gathering when she spotted the businessman wearing the

watch. She told him that he must either pay for the watch

or give it back. The businessman pulled back his jacket

to reveal the handle of a pistol that was tucked inside his

waistband. The clerk immediately walked away.

Of which of the following crimes should the

businessman be convicted?

(A) Robbery.

(B) Larceny.

(C) False pretenses.

(D) Embezzlement.

Question 59

A real estate agent was having an affair with his neighbor’s

wife. The neighbor learned of the affair and decided to

confront the real estate agent. The neighbor stormed over

to the real estate agent’s house carrying a loaded gun. The

real estate agent refused to open the door, instead yelling

at his neighbor through the closed door. Enraged at the real

estate agent’s refusal to open the door, the neighbor fired a

warning shot through the top of the real estate agent’s door.

The neighbor did not intend to injure the real estate agent

but simply to scare him. However, the door shattered and

several long, wooden shards flew off and became lodged

in the real estate agent’s chest and shoulder. The real estate

agent sued the neighbor for battery.

Who should prevail?

(A) The neighbor, because he did not intend for

the shards to strike the real estate agent.

(B) The neighbor, because he did not intend to

injure the real estate agent.

(C) The real estate agent, because the neighbor

failed to act as a reasonably prudent person.

(D) The real estate agent, because the neighbor

intended to shoot the gun.

Diagnostic Assessment

26

Question 60

The defendant, a citizen of Oklahoma, leased an

apartment from the plaintiff, a citizen of Idaho, under

a one-year rental agreement at $800 per month. The

lease stated that the defendant could not sublet the

premises to any person who is not a Christian. With six

months remaining on the lease, the defendant sublet

the apartment to a member of the Buddhist religion.

The plaintiff sued the defendant in the appropriate

U.S. District Court for breach of contract, seeking

to evict the defendant and recover the remaining six

months’ rent. The complaint stated that the defendant

violated the terms of the lease and asserts that the

clause concerning subletting does not violate the U.S.

Constitution. The defendant moves to dismiss the case

for lack of subject matter jurisdiction.

With respect to the defendant’s motion, will he prevail

in court on his motion to dismiss for lack of subject

matter jurisdiction?

(A) The defendant will prevail only with regard

to his assertion that subject matter jurisdiction

based upon a federal claim is lacking.

(B) The defendant will prevail only with regard

to his assertion that subject matter jurisdiction

based upon diversity is lacking.

(C) The defendant will prevail with regard to

his assertion that subject matter jurisdiction

based upon a federal claim is lacking and

with regard to his assertion that subject matter

jurisdiction based upon diversity is lacking.

(D) The defendant will not prevail in either assertion.

Question 61

One night, a woman returned from a long business trip

exhausted and ready to go to bed. However, she could not

find the keys to her house in her purse. The woman saw

that a window on the first floor of her neighbors’ house

was partially open. She did not know her neighbors well,

but she was aware that they were on vacation and did not

believe they would mind her crashing on their couch for

the night. The woman pushed the window open, crawled

through, and fell asleep on the neighbor’s couch. Just

READ ALSO :   What is the theme of chapter eight of Animal Farm?

before the neighbors arrived home, the woman awoke

and exited the house. As she was climbing back out the

window, she saw a snow globe that she liked and decided

to take it with her.

If the woman is prosecuted for burglary, how should

the court find?

(A) The woman is not guilty, because she did not

intend to commit a felony when she entered

the neighbor’s house.

(B) The woman is not guilty, because the house

was unoccupied when she entered it.

(C) The woman is guilty, because she broke into

and entered the dwelling of another person

during the night.

(D) The woman is guilty, because she took the snow

globe with the intent to permanently deprive.

Question 62

A buyer and a seller enter into a contract whereby the

seller would deliver to the buyer 300 cases of size

five spark plugs to be used in vehicles that the buyer

was producing at its plant. After the spark plugs were

delivered, the buyer wants to void the contract and

return the spark plugs.

What would be his best argument that he is not

required to pay for the spark plugs?

(A) The buyer lost its main vehicle purchaser

and no longer needs such a large quantity of

spark plugs.

(B) The spark plugs were size seven, instead of

size five.

(C) The contract was not signed by the seller.

(D) There was no price term included in the contract.

27

Question 63

A man was walking home from work late one night

when he was confronted by an acquaintance, who

demanded that the man pay him money owed from a

recent Super Bowl wager. The acquaintance threatened

the man and then proceeded to punch him in the face. As

the man was running away to escape the attack, a $100

dollar bill fell out of his pocket. After the man was gone,

the acquaintance glanced down and noticed the bill. The

acquaintance pocketed the bill and returned home.

For which of the following crimes should the

acquaintance be convicted?

(A) Robbery.

(B) Larceny.

(C) Battery and larceny.

(D) Battery.

Question 64

A man owned an independent grocery store in an

upscale urban area. The grocer prided himself on his

high-end deli meats and cheeses and his large selection

of expensive wines. The grocer had recently noticed,

however, that one of the freezers in the store leaked from

time to time, causing the tile underneath the freezer to

become wet. The grocer had called the freezer company

to repair the freezer, but the freezer company had not yet

sent a technician to repair the freezer.

While the grocer was preparing for the lunchtime rush, a

man came into the store and slipped on a portion of the

floor that was wet due to the freezer leakage. The man

landed awkwardly on his knee, shattering his kneecap.

If the man brings suit against the grocer, how should

the court rule?

(A) Judgment for the grocer, because by calling the

freezer company to fix the freezer, the grocer

took reasonable care to prevent the injury.

(B) Judgment for the man, but only if the grocer

knew that the leakage had expanded to the

area around the freezer.

(C) Judgment for the man, if the grocer could have

discovered the risk posed by the freezer leakage

by a reasonable inspection of his property.

(D) Judgment for the man, because the grocer is

liable for any condition on his premises that

injures a customer.

Question 65

A man leaves work very late one night and walks to his

car, which is parked in a parking garage. As the man nears

his car, he hears noises and what sounds like scuffling.

The man, who is an avid golfer, grabs a golf club from

his trunk and walks toward the area from where the noise

is coming. He sees a teenager with a knife in his hand,

kneeling over an unconscious woman on the ground. The

man recognizes the woman as his long-time secretary.

Believing that the teenager is about to stab the woman, the

man swings the golf club at the teenager, hitting him in

the head. The teenager dies from his injuries. It turns out

that the woman had had a seizure and the teenager was

going to use the knife to cut open the woman’s turtleneck

so she could breathe more easily.

Is the man guilty of murder for the teenager’s death?

(A) Yes, because the man intended to either kill or

seriously injure the teenager.

(B) Yes, because the man did not warn the

teenager before swinging the golf club.

(C) No, because the man reasonably believed

that his action was necessary to prevent the

stabbing of the woman.

(D) No, because the man knew the woman and

therefore had the right to use deadly force to

defend her.

Diagnostic Assessment

28

Question 66

A hiker came across a cat while hiking in the woods.

Not seeing an owner, the hiker decided to take the cat to

the local animal shelter. As she was carrying the cat into

the animal shelter, the hiker heard a man say, “I found

you!” The cat’s owner was so delighted to find his cat

that he promised to pay the hiker $250 for her troubles.

The hiker told him that that would not be necessary; she

had picked up the cat out of her love for animals and

expected no reward. Later, however, the hiker received

a large bill in the mail and realized that she could really

use the reward money. The hiker called the owner, but

the man refused to honor his earlier offer.

If the hiker sues to collect the reward, what will she recover?

(A) Nothing, because there was no consideration

for the reward.

(B) Nothing, because the hiker rejected the

man’s offer.

(C) $250, because the technical defense of the

Statute of Frauds will be overcome by the fact

that the hiker had fully performed.

(D) $250, because moral consideration is

sufficient to support the promise of a reward.

Question 67

A landlord and a tenant signed a lease agreement for

a one-year term. One month into the lease term, the

tenant changed jobs and abandoned the apartment

because his new job was an inconvenient commute

from the apartment. The landlord did not find a

replacement tenant. At the end of the lease term, the

landlord sent the tenant a bill for the unpaid rent.

In determining how much tenant owes, to which of the

following would most courts give the greatest weight?

(A) How burdensome the tenant’s new commute is.

(B) Whether the lease contained a covenant of

quiet enjoyment.

(C) Whether the landlord has met his burden

of showing that he attempted to find a

replacement tenant.

(D) Whether the tenant has met his burden of

showing that the landlord failed to attempt to

find a replacement tenant.

Question 68

A recently laid-off man was updating his resume on his

laptop computer at a local coffee shop. He noticed that

a woman at a nearby table was wearing huge diamond

earrings. When the woman took off her coat to sit

down, it caught on one of the earrings, and the earring

fell to the floor. The man realized that the earring

could probably be sold for the equivalent of several

months’ rent. He bent down as if to tie his shoe, and

then surreptitiously picked up the earring and placed

it in his computer bag. The woman soon realized that

her earring was missing. As she and her companion

searched for it, the man heard the woman mention

that the earrings had been a gift from her late father.

Feeling terrible, the man took the earring from his bag

and gave it to the woman. An elderly man who had

been watching the whole scene flagged down a passing

police officer and reported the incident.

Is the man guilty of larceny?

(A) Yes, because he took the earring and put it in

his bag, intending to keep it.

(B) Yes, because he fraudulently converted the earring.

(C) No, because he never left the coffee shop.

(D) No, because he gave the earring back to

the woman.

29

Question 69

A businesswoman with dual citizenship in the United

States and the United Kingdom decided to transfer her

entire business overseas. She entered into a contract with

a buyer for the sale of her home, which included a large

home office, for $325,000. The contract further provided

that the transfer of the deed, as well as the buyer’s

payment of the purchase price, would occur on May 1,

one month from the date of the contract. After the signing

of the contract, the businesswoman traveled to England to

begin the process of setting up her headquarters there. She

expected to stay four weeks and to return home the day

before the closing date of the contract. However, en route

home to the United States, the businesswoman found

herself stranded in Madrid, Spain, following a layover and

several flight delays and cancellations.

As a result, she did not arrive home until May 2,

the day after the closing date. Upon her return, the

businesswoman contacted the buyer to reschedule the

closing. However, the buyer was highly offended at the

businesswoman’s failure to show upon the promised date

or to contact him about the delay, and he informed her that

he would be canceling the contract due to her breach.

If the businesswoman files an action seeking the buyer’s

specific performance of the contract, will she prevail?

(A) No, because unless otherwise provided in

the terms of the contract, a party’s failure

to render performance on the date promised

constitutes a repudiation of the contract.

(B) No, because specific performance is not an

appropriate remedy for a buyer’s breach of a

contract for the sale of land.

(C) Yes, because the businesswoman’s delay

in performance was justified under the

circumstances.

(D) Yes, because the contract did not provide that

time was of the essence.

Question 70

Sunday was generally family day at the shopping mall.

One Sunday, two parents, like many others, took their

two children, a six-month-old daughter and a sevenyear-old

son, to the mall. At their son’s insistence,

the parents decided to let their son push the courtesy

stroller provided by the mall that was carrying the

daughter, while the father held the handle. The son

proudly pushed the stroller, grinning from ear to ear as

he walked along the main promenade.

The father’s former high school basketball coach smiled

as he passed the family. Ten minutes later, the coach

passed the family again. At that moment, the father let

go of the handle of the stroller to bend down to pick

up the daughter’s pacifier, and the son was temporarily

distracted. In his moment of distraction, the son ran the

stroller into the coach. The collision caused the coach to

fall to the ground, and he sustained a broken elbow.

If the coach sues the son, what is the son’s best defense?

(A) The coach assumed the risk when he saw the

son pushing the stroller.

(B) The son was exercising the care appropriate

for a reasonable child of like age, knowledge,

and experience.

(C) A seven-year-old is presumed incapable

of negligence.

(D) The son did not intend to strike the coach.

Diagnostic Assessment

30

Question 71

A plumber, who was a resident of California, was at a

convention in the State of Franklin. At the end of the

day, the plumber found what he thought was his car

in the hotel parking lot. Unbeknownst to the plumber,

his car key also opened up 10% of the other cars of

the same make and model. The plumber had in fact

taken the car of an off-duty security guard, a resident

of Franklin. The guard saw that the plumber was

driving off with his car, and he yelled at the plumber

hoping he would stop. When he did not, the guard

drew a handgun and shot the tires of his own car. The

plumber lost control of the car and crashed into the

front window of the hotel. The plumber was severely

injured in the crash and sustained over $80,000 in

medical bills. The plumber sued the guard in the

Federal District Court for the State of Franklin. The

plumber wishes to have a jury trial. The District Court,

following federal rules, grants a jury trial in such case

while Franklin state law would not.

Is the plumber entitled to a trial by jury?

(A) No, because the state law is outcome

determinative.

(B) No, because forum shopping is against

public policy.

(C) Yes, because the District Court has

proper jurisdiction.

(D) Yes, because the federal rule must be

followed in this case.

Question 72

One Saturday morning, a father took his ten-year-old

son to a little league game. The son, who played center

field, made several errors during the game. The coach

benched the son and berated him for costing the team

the game, driving the young boy to tears. The father

got into a heated argument with the coach over his

treatment of the boy. Bystanders eventually broke up

the argument, and the father drove his son home. Later

that night, while picking up take-out for the family’s

dinner, the father saw the coach in the restaurant’s

parking lot. Wanting to make the coach cry like the

coach had done to his son, the father grabbed his son’s

baseball bat from the back seat of his car, came up

behind the coach and hit him in the head with the bat.

The coach died from his injuries.

Is the father guilty of murder for the coach’s death?

(A) No, because the father acted in the heat of passion.

(B) No, because the father did not intend to kill

the coach.

(C) Yes, because a baseball bat may be a

dangerous instrumentality depending upon

how it is used.

(D) Yes, because, at a minimum, the father intended

to inflict serious injury upon the coach.

31

Question 73

A young entrepreneur opened a smoothie shop in a

thriving urban center. Wanting to provide plenty of seating

for his customers, the entrepreneur hired an independent

contractor to remodel the small space and build revolving

stools along all of the walls. One morning, shortly after

the grand opening of the shop, the lead cashier noticed

that one of the stools was loose, as if it might come

unbolted from the floor. The cashier immediately posted a

sign stating “Broken—Do Not Sit.”

The cashier then called the contractor and asked that

he come immediately to repair the stool. A few hours

later, a new customer entered the smoothie shop. While

waiting to give his order, the customer failed to notice the

cashier’s sign and sat directly on the loose stool. Under

the customer’s weight, the stool broke away from the

floor, and both the customer and the stool went crashing

to the ground. The customer broke his wrist in the fall.

If the customer sues the smoothie shop, will he recover?

(A) Yes, because the customer is a licensee.

(B) Yes, because the customer is a business invitee.

(C) No, because the contractor’s acts were the

actual and proximate cause of the injuries.

(D) No, because responsibility for construction of the

stool was properly delegated to the contractor.

Question 74

Apex became involved in a labor dispute with the

union that ordinarily supplied workers for Apex’s

plant. Apex commenced an action against the union in

the appropriate U.S. District Court claiming $75,000

in damages as a consequence of the union’s conduct

in harassing and intimidating nonunion workers in

violation of the National Labor Relations Act and

applicable state law. Apex is an Alabama corporation,

and the union (an unincorporated association) has

members who are domiciled in every state except

Washington and Oregon. The union answered by

denying Apex’s allegations and filing a $15,000 counterclaim

(which asserted that Apex had deliberately made

false accusations about the union to the local papers for

the purpose of obtaining favorable press coverage).

If the union moves to dismiss for lack of subject matter

jurisdiction, should the court grant the motion?

(A) Yes, because Apex has not claimed monetary

damages in excess of $75,000.

(B) Yes, because there is no diversity.

(C) No, because subject matter jurisdiction is satisfied.

(D) No, because a state claim has been asserted in

a federal court.

Diagnostic Assessment

32

Question 75

A buyer and seller entered into a written agreement

for the sale of 100 widgets at $1.50 each. The contract

provided that the widgets would be shipped in ten

cartons, each containing ten widgets. Because the

buyer knew that the cartons of widgets would be

moved several times before they were used, it was

important to him that the widgets be individually

wrapped. For this reason, the buyer requested that each

widget be individually packed and shipped in its own

carton. The seller agreed to this modification over the

phone. Shortly thereafter, the seller shipped the widgets

to the buyer in ten cartons, as originally agreed.

The buyer rejected the shipment and demanded new

widgets in individual cartons. The seller refused.

Is the seller in breach of contract?

(A) Yes, because contracts can always be modified.

(B) Yes, because the original contract can be orally

modified without additional consideration.

(C) No, because the modification must be in

writing when the sale of goods is involved.

(D) No, because there was no consideration for the

seller’s agreement to change the mode of shipping.

Question 76

In preparing for her law school examinations, a student

invited a few classmates to her apartment for a study

group. The student, who loved spiders, kept a pet

tarantula caged in her apartment. When the student

purchased the tarantula, she had the poisonous venom

removed so that it would be harmless.

During the study group session, the student brought out

the cage with the tarantula to show her friends. When

she was placing the cage down on the kitchen counter,

the student carelessly left the cage door slightly ajar.

Moments later, the tarantula crawled out of the cage

and crept toward one of the classmates. The classmate,

who was deathly afraid of spiders, saw the tarantula

and tried to run away. As she did so, the classmate

tripped over a chair and fell down, fracturing her wrist.

If the classmate files suit against the student to recover

damages for her injury, who will prevail?

(A) The classmate, because the student was

negligent in leaving the cage door open.

(B) The classmate, because the student is

strictly liable.

(C) The student, because the spider did not

directly cause the classmate’s injury.

(D) The student, because a spider is not a wild animal.

33

Question 77

In a state court of general jurisdiction, Paul sued the

Acme Corporation for personal injuries he received in

an auto collision with a van driven by an Acme driver.

Deborah was the president of Acme when the accident

occurred. Paul personally served Acme Corporation by

handing the summons and complaint to Deborah the day

before she retired. In the excitement of her retirement,

Deborah neglected to deliver the papers to anyone else

at Acme. A default was entered against Acme. Other

than service upon Deborah, Acme never received notice

of the pending lawsuit prior to the entry of a default

judgment. Acme now moves to quash service of process.

Applicable state law pertaining to service of process is

identical to the Federal Rules of Civil Procedure.

Which of the following is true?

(A) Service should be quashed, because the

summons and complaint were served by Paul.

(B) Service should be quashed, because Acme

Corporation did not receive actual notice of

the pending lawsuit.

(C) Service should be quashed, because Deborah

was not an officer of Acme at the time an

answer was due.

(D) Acme’s motion should be denied.

Question 78

A real estate tycoon owned four adjacent, equally sized

properties of undeveloped forest land in the mountains.

Each property consisted of 100 acres, and the properties

were respectively identified by the developer as the

northern property, southern property, eastern property, and

western property. One day, the developer took a buyer

out to view the properties and walked the buyer around

the wooded land, pointing out different aspects of each

property. Because it was high noon, the buyer became

confused about which direction he was facing while

looking at each of the properties. The buyer indicated that

if he decided to purchase any of the properties, he would

prefer the property that had the fewest trees, because it

would be cheaper to develop with fewer trees to clear.

The buyer mistakenly believed that this sparsely covered

property was the western property, and he looked forward

to having a view of the sunset from the living room of the

cabin he planned to build. The developer told the buyer

that the price of the property would be $300,000, payable

in two payments. The first payment of $150,000 would

be due within one month of agreeing to a deal, and the

second payment would be due six months later, when the

deed was to be executed.

A week later, the developer and buyer met for lunch to

confirm which property the buyer had selected, and the

buyer stated that he would like to purchase the western

property. The buyer and the developer shook hands on

the deal, and the developer promised to have a contract

sent to the buyer for signature. One week later, the buyer

received a contract signed by the developer, promising

to sell to the buyer the western property at the agreed

price and payment terms. Later that week, the buyer

went to visit the property with his contractors. Upon

arriving at the property, the buyer realized that he had

been mistaken in believing that the western property was

the property with the fewest trees; in fact, the property

with the fewest trees was the southern property. The

buyer tried to back out of the property contract, and the

developer sued to enforce the contract.

Can the developer enforce the contract to buy the property?

(A) No, because no part of the purchase price has

been paid.

(B) No, because the Statute of Frauds has not

been satisfied.

(C) Yes, because the agreement was put in writing.

(D) Yes, because the contract can be performed

within one year and is excluded from the

Statute of Frauds.

Diagnostic Assessment

34

Question 79

One evening, a fireman went bowling at a local bowling

alley. After he had finished bowling, he picked up and

put on what he believed to be his shoes, and went home.

About one hour later, as the fireman got ready for bed,

he discovered his mistake. Since it was nearly midnight

and the bowling alley was already closed, the fireman

decided to wait until the next day to return the shoes. The

following morning, the fireman was driving back to the

bowling alley with the shoes when he was involved in

a serious automobile accident. The shoes, which turned

out to be quite valuable, were destroyed. The actual

owner of the shoes, a handyman, then asserted a claim for

conversion against the fireman for the value of the shoes.

Who will prevail?

(A) The handyman, because the fireman intended

to take the shoes and they were destroyed

while in his possession.

(B) The handyman, because the fireman

committed a continuing trespass to a chattel

by failing to promptly return the shoes.

(C) The fireman, because he honestly believed the

shoes to be his own.

(D) The fireman, because he tried to return

the shoes within a reasonable time after

discovering his mistake.

Question 80

A father and his daughter bought a beach house as tenants

in common. They stayed in the beach house during their

many weekend trips to the beach to ski and surf. A few

years later, the father and his wife had another daughter.

The father died of a heart attack, leaving all his property

to his wife. Over the next 25 years, the older daughter

regularly used the beach house and declared to her mother

and younger sister that she was the true owner of the

beach house. When the older daughter decided to move

away, she rented the beach house to an avid surfer who

planned to live at the beach house full time. The surfer

paid the older sister $1,000 per month, which the older

sister deposited in her private bank account. The wife

then died, leaving all her property to her two daughters.

The younger daughter wants a share of the rent the older

daughter has been receiving.

If the younger sister sues the older sister for part of the

rent that the older sister is now collecting, what is the

likely outcome of the case?

(A) The younger sister will prevail, because

tenants in common are entitled to rent

received from third parties.

(B) The younger sister will prevail, because the

older sister’s declaration amounted to an ouster.

(C) The younger sister will not prevail, because

the older sister adversely possessed the

beach house.

(D) The younger sister will not prevail, because

the older sister had the right to exclusive use

of the beach house.

Academic help online